Does $lVert B^TA^{-1}BrVert_2<1$ imply that...











up vote
0
down vote

favorite












Does $leftlVert B^{operatorname{T}}A^{-1}BrightrVert_2<1$ imply that $leftlVertbegin{bmatrix}0&-A^{-1}B\0&-B^{operatorname{T}}A^{-1}Bend{bmatrix}rightrVert_2<1$, for $A,Binmathbb{R}^{Ntimes N}$ and $A=A^{operatorname{T}}>0$?



Or, at least, does $leftlVert B^{operatorname{T}}A^{-1}BrightrVert_2<1$ imply that $rholeft(begin{bmatrix}0&-A^{-1}B\0&-B^{operatorname{T}}A^{-1}Bend{bmatrix}right)<1$, where $rho(A)$ is the spectral radius of $A$?



I'm trying to solve a task where the first implication seems to me to be the sufficient condition for the thesis of the task to hold while the second implication seems to me to be equivalent (sufficient and necessary) to the thesis of the task.



I can't say much about the spectral radius... but while I can't prove it, to my intuition the implication that $leftlVert B^{operatorname{T}}A^{-1}BrightrVert_2<1impliesleftlVertbegin{bmatrix}0&-A^{-1}B\0&-B^{operatorname{T}}A^{-1}Bend{bmatrix}rightrVert_2<1$ is actually false. This perplexes me.



Does $leftlVert B^{operatorname{T}}A^{-1}BrightrVert_2<1$ imply any of the above conditions?










share|cite|improve this question




























    up vote
    0
    down vote

    favorite












    Does $leftlVert B^{operatorname{T}}A^{-1}BrightrVert_2<1$ imply that $leftlVertbegin{bmatrix}0&-A^{-1}B\0&-B^{operatorname{T}}A^{-1}Bend{bmatrix}rightrVert_2<1$, for $A,Binmathbb{R}^{Ntimes N}$ and $A=A^{operatorname{T}}>0$?



    Or, at least, does $leftlVert B^{operatorname{T}}A^{-1}BrightrVert_2<1$ imply that $rholeft(begin{bmatrix}0&-A^{-1}B\0&-B^{operatorname{T}}A^{-1}Bend{bmatrix}right)<1$, where $rho(A)$ is the spectral radius of $A$?



    I'm trying to solve a task where the first implication seems to me to be the sufficient condition for the thesis of the task to hold while the second implication seems to me to be equivalent (sufficient and necessary) to the thesis of the task.



    I can't say much about the spectral radius... but while I can't prove it, to my intuition the implication that $leftlVert B^{operatorname{T}}A^{-1}BrightrVert_2<1impliesleftlVertbegin{bmatrix}0&-A^{-1}B\0&-B^{operatorname{T}}A^{-1}Bend{bmatrix}rightrVert_2<1$ is actually false. This perplexes me.



    Does $leftlVert B^{operatorname{T}}A^{-1}BrightrVert_2<1$ imply any of the above conditions?










    share|cite|improve this question


























      up vote
      0
      down vote

      favorite









      up vote
      0
      down vote

      favorite











      Does $leftlVert B^{operatorname{T}}A^{-1}BrightrVert_2<1$ imply that $leftlVertbegin{bmatrix}0&-A^{-1}B\0&-B^{operatorname{T}}A^{-1}Bend{bmatrix}rightrVert_2<1$, for $A,Binmathbb{R}^{Ntimes N}$ and $A=A^{operatorname{T}}>0$?



      Or, at least, does $leftlVert B^{operatorname{T}}A^{-1}BrightrVert_2<1$ imply that $rholeft(begin{bmatrix}0&-A^{-1}B\0&-B^{operatorname{T}}A^{-1}Bend{bmatrix}right)<1$, where $rho(A)$ is the spectral radius of $A$?



      I'm trying to solve a task where the first implication seems to me to be the sufficient condition for the thesis of the task to hold while the second implication seems to me to be equivalent (sufficient and necessary) to the thesis of the task.



      I can't say much about the spectral radius... but while I can't prove it, to my intuition the implication that $leftlVert B^{operatorname{T}}A^{-1}BrightrVert_2<1impliesleftlVertbegin{bmatrix}0&-A^{-1}B\0&-B^{operatorname{T}}A^{-1}Bend{bmatrix}rightrVert_2<1$ is actually false. This perplexes me.



      Does $leftlVert B^{operatorname{T}}A^{-1}BrightrVert_2<1$ imply any of the above conditions?










      share|cite|improve this question















      Does $leftlVert B^{operatorname{T}}A^{-1}BrightrVert_2<1$ imply that $leftlVertbegin{bmatrix}0&-A^{-1}B\0&-B^{operatorname{T}}A^{-1}Bend{bmatrix}rightrVert_2<1$, for $A,Binmathbb{R}^{Ntimes N}$ and $A=A^{operatorname{T}}>0$?



      Or, at least, does $leftlVert B^{operatorname{T}}A^{-1}BrightrVert_2<1$ imply that $rholeft(begin{bmatrix}0&-A^{-1}B\0&-B^{operatorname{T}}A^{-1}Bend{bmatrix}right)<1$, where $rho(A)$ is the spectral radius of $A$?



      I'm trying to solve a task where the first implication seems to me to be the sufficient condition for the thesis of the task to hold while the second implication seems to me to be equivalent (sufficient and necessary) to the thesis of the task.



      I can't say much about the spectral radius... but while I can't prove it, to my intuition the implication that $leftlVert B^{operatorname{T}}A^{-1}BrightrVert_2<1impliesleftlVertbegin{bmatrix}0&-A^{-1}B\0&-B^{operatorname{T}}A^{-1}Bend{bmatrix}rightrVert_2<1$ is actually false. This perplexes me.



      Does $leftlVert B^{operatorname{T}}A^{-1}BrightrVert_2<1$ imply any of the above conditions?







      matrices block-matrices spectral-radius matrix-norms






      share|cite|improve this question















      share|cite|improve this question













      share|cite|improve this question




      share|cite|improve this question








      edited yesterday









      user10354138

      6,274623




      6,274623










      asked yesterday









      gaazkam

      422313




      422313






















          1 Answer
          1






          active

          oldest

          votes

















          up vote
          2
          down vote



          accepted










          The answer to your first question is no. The answer to your second question is yes.



          If $N=1$ then this is equivalent to asking whether $b^2/a<1$ implies
          $$left|begin{bmatrix}0&-b/a\0&-b^2/aend{bmatrix}right|_2<1,$$
          where $binmathbb{R}$ and $ain(0,infty)$.



          But this seems false. Take $a=1/2$ and $b=2/3$. Then $b^2/a=8/9<1$, however
          $$left|begin{bmatrix}0&-b/a\0&-b^2/aend{bmatrix}right|_2=left|begin{bmatrix}0&-4/3\0&-8/9end{bmatrix}right|_2=frac{4sqrt{13}}{9}>1.$$



          Probably this counterexample can be extended to all $Ninmathbb{N}$ by letting $A=frac{1}{2}I_N$ and $B=frac{2}{3}I_N$.



          On the other hand, for $N=1$ the above matrix will always have spectrum ${-b^2/a,0}$. So the spectral radius will be $b^2/a<1$.



          The case for $N>1$ requires the block matrix property of determinants:
          $$text{det}begin{pmatrix}A&B\0&Dend{pmatrix}=text{det}(A)text{det}(D).$$
          Observe that the eigenvalues to your matrix are the solutions to
          $$0=text{det}begin{pmatrix}-lambda&-A^{-1}B\0&-B^TA^{-1}B-lambdaend{pmatrix}=text{det}(-lambda)text{det}(-B^TA^{-1}B-lambda)$$
          which is true precisely when $text{det}(-lambda)=0$ or else $text{det}(-B^TA^{-1}B-lambda)=0$. The former implies $lambda=0$, while the latter gives the negative eigenvalues to $B^TA^{-1}B$. It is well-known that the spectral radius is bounded above by the 2-norm. Hence
          $$rhobegin{pmatrix}0&-A^{-1}B\0&-B^TA^{-1}Bend{pmatrix}=rho(B^TA^{-1}B)leq|B^TA^{-1}B|_2<1.$$






          share|cite|improve this answer























            Your Answer





            StackExchange.ifUsing("editor", function () {
            return StackExchange.using("mathjaxEditing", function () {
            StackExchange.MarkdownEditor.creationCallbacks.add(function (editor, postfix) {
            StackExchange.mathjaxEditing.prepareWmdForMathJax(editor, postfix, [["$", "$"], ["\\(","\\)"]]);
            });
            });
            }, "mathjax-editing");

            StackExchange.ready(function() {
            var channelOptions = {
            tags: "".split(" "),
            id: "69"
            };
            initTagRenderer("".split(" "), "".split(" "), channelOptions);

            StackExchange.using("externalEditor", function() {
            // Have to fire editor after snippets, if snippets enabled
            if (StackExchange.settings.snippets.snippetsEnabled) {
            StackExchange.using("snippets", function() {
            createEditor();
            });
            }
            else {
            createEditor();
            }
            });

            function createEditor() {
            StackExchange.prepareEditor({
            heartbeatType: 'answer',
            convertImagesToLinks: true,
            noModals: true,
            showLowRepImageUploadWarning: true,
            reputationToPostImages: 10,
            bindNavPrevention: true,
            postfix: "",
            imageUploader: {
            brandingHtml: "Powered by u003ca class="icon-imgur-white" href="https://imgur.com/"u003eu003c/au003e",
            contentPolicyHtml: "User contributions licensed under u003ca href="https://creativecommons.org/licenses/by-sa/3.0/"u003ecc by-sa 3.0 with attribution requiredu003c/au003e u003ca href="https://stackoverflow.com/legal/content-policy"u003e(content policy)u003c/au003e",
            allowUrls: true
            },
            noCode: true, onDemand: true,
            discardSelector: ".discard-answer"
            ,immediatelyShowMarkdownHelp:true
            });


            }
            });














             

            draft saved


            draft discarded


















            StackExchange.ready(
            function () {
            StackExchange.openid.initPostLogin('.new-post-login', 'https%3a%2f%2fmath.stackexchange.com%2fquestions%2f3004930%2fdoes-lvert-bta-1b-rvert-21-imply-that-left-lvert-beginbmatrix0-a%23new-answer', 'question_page');
            }
            );

            Post as a guest















            Required, but never shown

























            1 Answer
            1






            active

            oldest

            votes








            1 Answer
            1






            active

            oldest

            votes









            active

            oldest

            votes






            active

            oldest

            votes








            up vote
            2
            down vote



            accepted










            The answer to your first question is no. The answer to your second question is yes.



            If $N=1$ then this is equivalent to asking whether $b^2/a<1$ implies
            $$left|begin{bmatrix}0&-b/a\0&-b^2/aend{bmatrix}right|_2<1,$$
            where $binmathbb{R}$ and $ain(0,infty)$.



            But this seems false. Take $a=1/2$ and $b=2/3$. Then $b^2/a=8/9<1$, however
            $$left|begin{bmatrix}0&-b/a\0&-b^2/aend{bmatrix}right|_2=left|begin{bmatrix}0&-4/3\0&-8/9end{bmatrix}right|_2=frac{4sqrt{13}}{9}>1.$$



            Probably this counterexample can be extended to all $Ninmathbb{N}$ by letting $A=frac{1}{2}I_N$ and $B=frac{2}{3}I_N$.



            On the other hand, for $N=1$ the above matrix will always have spectrum ${-b^2/a,0}$. So the spectral radius will be $b^2/a<1$.



            The case for $N>1$ requires the block matrix property of determinants:
            $$text{det}begin{pmatrix}A&B\0&Dend{pmatrix}=text{det}(A)text{det}(D).$$
            Observe that the eigenvalues to your matrix are the solutions to
            $$0=text{det}begin{pmatrix}-lambda&-A^{-1}B\0&-B^TA^{-1}B-lambdaend{pmatrix}=text{det}(-lambda)text{det}(-B^TA^{-1}B-lambda)$$
            which is true precisely when $text{det}(-lambda)=0$ or else $text{det}(-B^TA^{-1}B-lambda)=0$. The former implies $lambda=0$, while the latter gives the negative eigenvalues to $B^TA^{-1}B$. It is well-known that the spectral radius is bounded above by the 2-norm. Hence
            $$rhobegin{pmatrix}0&-A^{-1}B\0&-B^TA^{-1}Bend{pmatrix}=rho(B^TA^{-1}B)leq|B^TA^{-1}B|_2<1.$$






            share|cite|improve this answer



























              up vote
              2
              down vote



              accepted










              The answer to your first question is no. The answer to your second question is yes.



              If $N=1$ then this is equivalent to asking whether $b^2/a<1$ implies
              $$left|begin{bmatrix}0&-b/a\0&-b^2/aend{bmatrix}right|_2<1,$$
              where $binmathbb{R}$ and $ain(0,infty)$.



              But this seems false. Take $a=1/2$ and $b=2/3$. Then $b^2/a=8/9<1$, however
              $$left|begin{bmatrix}0&-b/a\0&-b^2/aend{bmatrix}right|_2=left|begin{bmatrix}0&-4/3\0&-8/9end{bmatrix}right|_2=frac{4sqrt{13}}{9}>1.$$



              Probably this counterexample can be extended to all $Ninmathbb{N}$ by letting $A=frac{1}{2}I_N$ and $B=frac{2}{3}I_N$.



              On the other hand, for $N=1$ the above matrix will always have spectrum ${-b^2/a,0}$. So the spectral radius will be $b^2/a<1$.



              The case for $N>1$ requires the block matrix property of determinants:
              $$text{det}begin{pmatrix}A&B\0&Dend{pmatrix}=text{det}(A)text{det}(D).$$
              Observe that the eigenvalues to your matrix are the solutions to
              $$0=text{det}begin{pmatrix}-lambda&-A^{-1}B\0&-B^TA^{-1}B-lambdaend{pmatrix}=text{det}(-lambda)text{det}(-B^TA^{-1}B-lambda)$$
              which is true precisely when $text{det}(-lambda)=0$ or else $text{det}(-B^TA^{-1}B-lambda)=0$. The former implies $lambda=0$, while the latter gives the negative eigenvalues to $B^TA^{-1}B$. It is well-known that the spectral radius is bounded above by the 2-norm. Hence
              $$rhobegin{pmatrix}0&-A^{-1}B\0&-B^TA^{-1}Bend{pmatrix}=rho(B^TA^{-1}B)leq|B^TA^{-1}B|_2<1.$$






              share|cite|improve this answer

























                up vote
                2
                down vote



                accepted







                up vote
                2
                down vote



                accepted






                The answer to your first question is no. The answer to your second question is yes.



                If $N=1$ then this is equivalent to asking whether $b^2/a<1$ implies
                $$left|begin{bmatrix}0&-b/a\0&-b^2/aend{bmatrix}right|_2<1,$$
                where $binmathbb{R}$ and $ain(0,infty)$.



                But this seems false. Take $a=1/2$ and $b=2/3$. Then $b^2/a=8/9<1$, however
                $$left|begin{bmatrix}0&-b/a\0&-b^2/aend{bmatrix}right|_2=left|begin{bmatrix}0&-4/3\0&-8/9end{bmatrix}right|_2=frac{4sqrt{13}}{9}>1.$$



                Probably this counterexample can be extended to all $Ninmathbb{N}$ by letting $A=frac{1}{2}I_N$ and $B=frac{2}{3}I_N$.



                On the other hand, for $N=1$ the above matrix will always have spectrum ${-b^2/a,0}$. So the spectral radius will be $b^2/a<1$.



                The case for $N>1$ requires the block matrix property of determinants:
                $$text{det}begin{pmatrix}A&B\0&Dend{pmatrix}=text{det}(A)text{det}(D).$$
                Observe that the eigenvalues to your matrix are the solutions to
                $$0=text{det}begin{pmatrix}-lambda&-A^{-1}B\0&-B^TA^{-1}B-lambdaend{pmatrix}=text{det}(-lambda)text{det}(-B^TA^{-1}B-lambda)$$
                which is true precisely when $text{det}(-lambda)=0$ or else $text{det}(-B^TA^{-1}B-lambda)=0$. The former implies $lambda=0$, while the latter gives the negative eigenvalues to $B^TA^{-1}B$. It is well-known that the spectral radius is bounded above by the 2-norm. Hence
                $$rhobegin{pmatrix}0&-A^{-1}B\0&-B^TA^{-1}Bend{pmatrix}=rho(B^TA^{-1}B)leq|B^TA^{-1}B|_2<1.$$






                share|cite|improve this answer














                The answer to your first question is no. The answer to your second question is yes.



                If $N=1$ then this is equivalent to asking whether $b^2/a<1$ implies
                $$left|begin{bmatrix}0&-b/a\0&-b^2/aend{bmatrix}right|_2<1,$$
                where $binmathbb{R}$ and $ain(0,infty)$.



                But this seems false. Take $a=1/2$ and $b=2/3$. Then $b^2/a=8/9<1$, however
                $$left|begin{bmatrix}0&-b/a\0&-b^2/aend{bmatrix}right|_2=left|begin{bmatrix}0&-4/3\0&-8/9end{bmatrix}right|_2=frac{4sqrt{13}}{9}>1.$$



                Probably this counterexample can be extended to all $Ninmathbb{N}$ by letting $A=frac{1}{2}I_N$ and $B=frac{2}{3}I_N$.



                On the other hand, for $N=1$ the above matrix will always have spectrum ${-b^2/a,0}$. So the spectral radius will be $b^2/a<1$.



                The case for $N>1$ requires the block matrix property of determinants:
                $$text{det}begin{pmatrix}A&B\0&Dend{pmatrix}=text{det}(A)text{det}(D).$$
                Observe that the eigenvalues to your matrix are the solutions to
                $$0=text{det}begin{pmatrix}-lambda&-A^{-1}B\0&-B^TA^{-1}B-lambdaend{pmatrix}=text{det}(-lambda)text{det}(-B^TA^{-1}B-lambda)$$
                which is true precisely when $text{det}(-lambda)=0$ or else $text{det}(-B^TA^{-1}B-lambda)=0$. The former implies $lambda=0$, while the latter gives the negative eigenvalues to $B^TA^{-1}B$. It is well-known that the spectral radius is bounded above by the 2-norm. Hence
                $$rhobegin{pmatrix}0&-A^{-1}B\0&-B^TA^{-1}Bend{pmatrix}=rho(B^TA^{-1}B)leq|B^TA^{-1}B|_2<1.$$







                share|cite|improve this answer














                share|cite|improve this answer



                share|cite|improve this answer








                edited yesterday

























                answered yesterday









                Ben W

                1,079510




                1,079510






























                     

                    draft saved


                    draft discarded



















































                     


                    draft saved


                    draft discarded














                    StackExchange.ready(
                    function () {
                    StackExchange.openid.initPostLogin('.new-post-login', 'https%3a%2f%2fmath.stackexchange.com%2fquestions%2f3004930%2fdoes-lvert-bta-1b-rvert-21-imply-that-left-lvert-beginbmatrix0-a%23new-answer', 'question_page');
                    }
                    );

                    Post as a guest















                    Required, but never shown





















































                    Required, but never shown














                    Required, but never shown












                    Required, but never shown







                    Required, but never shown

































                    Required, but never shown














                    Required, but never shown












                    Required, but never shown







                    Required, but never shown







                    Popular posts from this blog

                    Can a sorcerer learn a 5th-level spell early by creating spell slots using the Font of Magic feature?

                    ts Property 'filter' does not exist on type '{}'

                    Notepad++ export/extract a list of installed plugins